LSAT and Law School Admissions Forum

Get expert LSAT preparation and law school admissions advice from PowerScore Test Preparation.

 Administrator
PowerScore Staff
  • PowerScore Staff
  • Posts: 8917
  • Joined: Feb 02, 2011
|
#101052
Complete Question Explanation

Weaken. The correct answer choice is (B).

Answer choice (A):

Answer choice (B): This is the correct answer choice.

Answer choice (C):

Answer choice (D):

Answer choice (E):
 ChicaRosa
  • Posts: 111
  • Joined: Aug 23, 2016
|
#30821
I was stuck between B and C and ended up choosing C. I want to know why C is incorrect while B is correct?

I thought C was correct because the argument mentions that the average discretionary income of those over 65 will soon be greater than that of young adults. I thought B neither strengthened nor weakened the argument since I didn't understand how it would weaken the argument.

Thanks!
 Adam Tyson
PowerScore Staff
  • PowerScore Staff
  • Posts: 5153
  • Joined: Apr 14, 2011
|
#31259
Remember to focus on the conclusion, Chica, for that's what we are trying to weaken (or strengthen, or justify, as the case may be). What is the author trying to prove? It's not that the older folks will have more discretionary income, because that is a premise, something he takes for granted and we are supposed to accept as true. Rather, the conclusion, based on that and other premises, is that, in the future, television advertisers would do better to target the older folks instead of the younger ones.

Answer B tells us that targeting the older folks is probably a waste of time. The goal is to create brand loyalty (see the very first line of the stimulus), so if older folks already have those loyalties in place, it's probably too late to achieve that goal.

Answer C doesn't hurt, because while it may be true that younger folks will soon have more discretionary income than they currently have, the premise tells us that older folks will have even more than the younger folks will have. Young folks may be seeing an increase, but older folks will see and even greater increase, to the point that they will surpass the younger ones. We still accept the truth of the premise, and C doesn't do anything to undermine that premise.

See if that makes sense, and give it another go. Don't lose sight of the conclusion, and remember to prephrase your answer so you won't fall for attractive wrong answers like C.

Good luck!

Get the most out of your LSAT Prep Plus subscription.

Analyze and track your performance with our Testing and Analytics Package.